XÂY DỰNG QUY NẠ P TRUY HỒ

Một phần của tài liệu Bộ đề thi năng khiếu môn toán lớp 10 trường chuyên năm 2022 (Trang 87 - 90)

- Nế uT thuộc tập K, thì A lại có thể chọn ngườ i1 bắt cặp với T, quá trình cứ lặp lại như vậy rồi sẽ đến lúc B phải chọn được một người “T” nào đó không thuộc tập

2.5.XÂY DỰNG QUY NẠ P TRUY HỒ

2. MỘT SỐ PHƯƠNG PHÁP GIẢI TOÁN CỰC TRỊ TẬP HỢP

2.5.XÂY DỰNG QUY NẠ P TRUY HỒ

Ví dụ 2.5.1. Chonnguyên dương và tập hợpM={1,2, . . . ,20}. GọiA1,A2, . . . ,Anlà các tập con phân

biệt khác rỗng củaMsao cho

|AiAj| ≤2,∀1≤i< jn. Tìm giá trị lớn nhất củan.

Chứng minh. 1. Giả sửA1,A2, . . . ,An là các tập con của M thỏa mãn điều kiện bài toán. Nếu một trong các tậpA1,A2, . . . ,Ancó ít nhất 4 phần tử, không mất tính tổng quát, giả sử|A1| ≥4. Gọia

là một phần tử trongA1.

• Khi đóA1\{a}là tập có ít nhất ba phần tử. Do|A1∩Ai| ≤2,∀i=2,3, . . . ,nnên tậpA1\{a}

phải phân biệt với tất cả các tậpA2, . . . ,An (vì nếuA1\{a} ≡Ainào đó, thì A1\{a} ∩Ai=A1\{a}

có ít nhất ba phần tử. Khi đóA1∩Ai cũng có ít nhất ba phần tử, vô lý.)

CỰC TRỊ TẬP HỢP 2 MỘT SỐ PHƯƠNG PHÁP GIẢI TOÁN CỰC TRỊ TẬP HỢP

• Ngoài ra

|(A1\{a})∩Aj| ≤ |A1∩Aj| ≤2,∀j =2,3, . . . ,n. 2. Từ các kết quả trên, nếu thayA1 bởiA1\{a}thì hệ tập hợp

A1\{a},A2,A3, . . . ,An

vẫn thỏa mãn bài toán. Hệ mới này có cùng số tập hợp với hệ ban đầu, nhưng số phần tử trong

mỗi tập hợp ≤1so với số phần tử trong mỗi tập hợp của hệ ban đầu.Cứ tiếp tục quá trình này,

ta thu được dãy tập hợpA∗1, . . . ,Anthỏa mãn bài toán và

|Ai| ≤3,∀i=1,2, . . . ,n

Tức là các tập hợp này, mỗi tập hợp chỉ có tối đa ba phần tử. Do đó

n≤ 20 1 + 20 2 + 20 3 =1350.

3. Tiếp theo, tổng số các tập con chứa 1 phần tử, chứa 2 phần tử, chứa 3 phần tử của M

20 1 + 20 2 + 20 3 =1350

và tất cả các tập này thỏa mãn điều kiện bài toán (rõ ràng hai tập phân biệt bất kỳ trong các tập này có giao không vượt quá 2 phần tử). Do đó n=1350đạt được.

Từ đó suy ra giá trị lớn nhất củann=1350.

Ví dụ 2.5.2(Indian 2014). Cho số tự nhiênnX ={1,2, . . . ,n}. Với hai tập conABcủa X, ký

hiệu

AB={iX|i∈(A\B)∪(B\A)}.

GọiF là một họ các tập con củaX sao cho với mọiA,B∈F thì

|AB| ≥2.

Chứng minh rằng|F| ≤2n−1 và tìm tất cả các họF có2n−1 phần tử.

Chứng minh. 1. Với mỗi tập con A⊂ {1,2, . . . ,n−1}, thì trong cặp tập hợp (A,A∪ {n}), tối đa một tập hợp thuộc vàoF. Thật vậy vì

A\(A∪ {n}) = /0, (A∪ {n})\A={n} ⇒A∆(A∪ {n}) ={n}.

Mặt khác, ta có tối đa 2n−1cặp tập hợp (A,A∪ {n}). Do đó tậpF có tối đa2n−1phần tử. 2. Tiếp theo ta chứng minh bằng quy nạp theonlà nếu|F|=2n−1 thìF hoặc chứa tất cả các tập

con chứa số lẻ phần tử hoặcF chứa tất cả các tập hợp chứa số lẻ phần tử.

• Kết quả hiển nhiên đúng vớin=1.

CỰC TRỊ TẬP HỢP 2 MỘT SỐ PHƯƠNG PHÁP GIẢI TOÁN CỰC TRỊ TẬP HỢP

• Giả sử kết quả đúng đến n=m−1, với m nguyên dương lớn hơn 1. Xét trường hợp với

n=m. Đặt

F1={A∈F|mA}, F2={A∈F|m6∈A}. Theo giả thiết quy nạp, tậpF∈có nhiều nhất2m−2 phần tử.

• Với mỗi tập hợpA∈F1, ta xét tập hợp mớiA\{m}. Khi đó

F3 ={A\{m}|A∈F1}

Khi đó tập F3 cũng thỏa mãn yêu cầu bài toán và theo giả thiết quy nạp thì lại có|F3| ≤

2m−2. Nhưng

|F2|=|F3| ≤2m−2⇒ |F|=|F1|+|F2| ≤2m−2+2m−2=2m−1.

Do đó nếu |F|=2m−1 thì |F1| =|F2|=2m−2. Nhưng khi đó lại theo giả thiết quy nạp, họF∈chứa tất cả các tập con của\1,2, . . . ,m−1với, giả sử, số chẵn phần tử. Khi đóF∞ chứa tất cả các tập con của {1,2, . . . ,m}, chứa m, dạng A∪ {m}. Tuy nhiên A không thể thuộc vàoF2, vì nếu không thì cảA,A∪ {m}đều thuộc vàoF vô lý. Khi đóAphải có số lẻ phần tử,A⊂ {1,2, . . . ,m−1}, và|F1|=2m−2 nênAphải chạy trên tất cả các tập có số

lẻ phần tử. Do đóF1cũng chứa các tập có số chẵn phần tử. VậyF chứa tất cả các tập hợp

có số chẵn phần tử. Tương tự khiF2chứa tất cả các tập có số lẻ phần tử. Bài toán được chứng minh.

Ví dụ 2.5.3(Bankal 2012). Chonlà số nguyên dương. Đặt tập hợpPn={2n,2n−1.3,2n−2.32, . . . ,3n}.

Với mỗi tập conX củaPn, đặtSX là tổng tất cả các phần tử trongX, ở đâyS/0=0. Choylà một số thực thỏa mãn điều kiện0≤y≤3n+1−2n+1. Chứng minh rằng tồn tại một tập conY củaPnthỏa mãn điều kiện0≤ySY <2n.

Giải. Ta có

SPn =3n+3n−1.2+···+32.2n−2+3.2n−1+2n

= (3−2)(3n+3n−1.2+···+32.2n−2+3.2n−1+2n) =3n+1−2n+1

Bằng cách chia cách phần tử củaPn cho2n ta đưa bài toán về dạng tương đương như sau:

Chonlà số nguyên dương,a= 3

2, vàQn={1,a,a2, . . . ,an}. Chứng tỏ rằng với mỗi giá trị củaxthỏa mãn0≤x≤1+a+a2+···+an, luôn tồn tại một tập conX củaQn sao cho0≤xSX <1.

Ta chứng minh bằng quy nạp theon. Khin=1thìS/0=0,S{1}=1,S{a}= 3

2,S{1,a} = 5

2. Từ đây kiểm chứng thấy ngay là nếuxlà một số thực thỏa0≤x≤ 52 thì luôn có một tập conX củaQ1thỏa yêu cầu.

Giả sử kết quả bài toán đúng đến số nguyên dươngn.

Xétxlà một số thực với0≤x≤1+a+a2+···+an+an+1.

+ Nếu 0≤x≤1+a+a2+···+an thì theo giả thiết quy nạp tồn tại tập con XQnQm+1 thỏa

Một phần của tài liệu Bộ đề thi năng khiếu môn toán lớp 10 trường chuyên năm 2022 (Trang 87 - 90)